-4

Let all rational numbers qn of the real axis be covered by intervals In of measure |In|=2n, such that qn is the centre of In. Then the endpoints are rational numbers. The irrational numbers x of the complement of infinite measure, not covered by the intervals In, form particles of a totally disconnected space, so-called "Cantor dust" Ξ.

Every particle xΞ must be separated from every particle yΞ by at least one rational number qn and hence by at least one interval In covering qn. Since the end points of the In are rational numbers too, also being covered by their own intervals, the particles of Cantor dust can only be limits of infinite sequences of overlapping intervals In.

If intervals don't overlap, then they cannot form a common limit; their limits must lie between them. But in any case infinitely many finite intervals are required to establish one limit. And in any case two limits must be separated by infinitely many overlapping intervals

Such an infinite set of overlapping intervals is called a cluster. In principle, given a fixed and complete enumeration of the rationals, we can calculate every cluster and the limits of its union. Therefore, every irrational xΞ can be put in bijection with the pair of clusters, i.e., the infinite set of intervals converging to it. There are countably many sets In and therefore not more than countably many disjoint clusters with limits and therefore not more limits.

Where are the other irrational numbers of the complement?

EDIT: Maybe that the closers thought to have the Cantor-sets as a counterexample. While Cantor-sets are based on the same fallacy of set theory, namely the unjustified extrapolation from finite to infinite bijections, they are not as easily to refute because there is not the crucial condition that irrational points must be separated by clusters of finite measure.

New contributor
Ganzhinterseher is a new contributor to this site. Take care in asking for clarification, commenting, and answering. Check out our Code of Conduct.

put on hold as off-topic by Alexandre Eremenko, Joel David Hamkins, Noah Schweber, Andrés E. Caicedo, Jan-Christoph Schlage-Puchta 21 hours ago

This question appears to be off-topic for this site. While what’s on- and off-topic is not always intuitive, you can learn more about it by reading the help center. The users who voted to close gave this specific reason:

  • "This question does not appear to be about research level mathematics within the scope defined in the help center." – Alexandre Eremenko, Joel David Hamkins, Noah Schweber, Andrés E. Caicedo, Jan-Christoph Schlage-Puchta
If this question can be reworded to fit the rules in the help center, please edit your question.

  • This question is more appropriate for Mathematics Stack Exchange, although it has already been asked there in many different ways, such as Is the Cantor set made of interval endpoints? AND Is every point in the Cantor set rational?. – Dave L Renfro yesterday
  • This question has nothing to do with the Cantor set. The common answer is: "N finite intervals leave N+1 parts in the complement but as soon as you have an infinite set, weird things can (and do) happen." Of course this wisdom is correct. But it has to be appplied in the first place to bijections between infinite sets and therefore can solve all problems connected with transfinite set theory by showing that its basis is invalid. – Ganzhinterseher yesterday  
  • nothing to do with the Cantor set --- This is still more appropriate for MSE. Also, you should edit your question to avoid the (presumably) natural language use of words having generally accepted mathematical meanings and the refurbishing of mathematical terms without giving precise definitions, such as Cantor dust, particle (do you mean element?), finite interval, complete enumeration, (continued) – Dave L Renfro yesterday
  • I know all that. It does not explain anything about clusters of intervals. There is only the the wisdom that I quoted already and that is true. Infinite sets behave differently from finite sets. For instance: If we remove N intervals from the real axis, then N+1 intervals will remain. But if we remove countably many intervals, then uncountably many atoms or degenerate intervals will remain. But the most important application is rarely mentioned: If we biject finite sets, then we get information about their relative cardinality. For infinite sets this method fails. Same with the diagonal – Ganzhinterseher yesterday  
  • argument. There is no information obtained. Otherwise the intervals or clusters could be used as an enumeration of the irrationals they converge to. Their number does not change when shuffled. But the method itself is invalid - since infinite sets behave differently. – Ganzhinterseher yesterday  
  • @Dave L Renfro: Have you never wondered why the bijection between every q and every n is constructed for every pair (q, n) in a finite set (since every n belongs to a finite initial segment) but is claimed to be valid for the infinite sets? Injectivity and surjectivity can be proven for every q and n. But all belong to a finite initial segment. Nothing is proven and nothing can be proven for the whole infinite sets. We know: Infinite sets behave differently. – Ganzhinterseher yesterday  
  • The mistake is the assumption that converging intervals must overlap. Take an interval of length 2n around m/n, and look what happens for any poorly approximable number. – Jan-Christoph Schlage-Puchta 5 hours ago
  • If converging intervals do not overlap, then they converge to the gap between them. Clusters are defined as path-connected set of intervals with two irrational limits or endpoints. Given the complete enumeration every limit can be calculated. Every limit however is the endpoint of two clusters. Otherwise there was a gap of only irrational numbers contradicting mathematics. Poorly approximatable numbers merely show that the assumption of the existence of as many rationals as naturals is wrong. – Ganzhinterseher 4 hours ago  
  • A point of the complement disconnects two clusters or it has, at least in one direction, a finite distance from every interval and a neigbourhood containing only irrational points of the complement. – Ganzhinterseher 3 hours ago